Help with a real analysis problem

Click For Summary
The discussion centers on proving a concept in real analysis, specifically regarding the existence of a measure ## \mu'##. The initial approach involved attempting a proof by contradiction but was deemed ineffective. Instead, the focus shifted to starting with the definition of f being in L^1, which allows for the construction of a sequence of step functions to approximate f. This approximation can help define sets of finite measure that modify the measure ## \mu##. A direct proof approach is recommended over contradiction for clarity and effectiveness.
polkadot66
Messages
1
Reaction score
0
Homework Statement
Given ## f \in L^1(X, M, \mu)## , show there is a ## \sigma## -finite measure ## \mu'## such that ## \int_{E} f d\mu = \int_{E} f d\mu'## .
Relevant Equations
## f \in L^1## so ## \int |f| d\mu < \infty##
a measure ## \mu'## is ## \sigma##-finite if there are sets ##A_1, A_2, ...## such that ## \mu'(A_n) < \infty## and ##X= \cup A_n##
I tried to prove this by absurd stating that there is no such ## \mu'## but i couldn't get anywhere...
 
Physics news on Phys.org
i would begin with the definition of f being in L^1. That should give you a sequence of step functions that approximate f and can be used to define some sets of finite measure that can be used to modify mu. anyway i would do it directly, not by contradiction.
 
Question: A clock's minute hand has length 4 and its hour hand has length 3. What is the distance between the tips at the moment when it is increasing most rapidly?(Putnam Exam Question) Answer: Making assumption that both the hands moves at constant angular velocities, the answer is ## \sqrt{7} .## But don't you think this assumption is somewhat doubtful and wrong?

Similar threads

  • · Replies 21 ·
Replies
21
Views
1K
  • · Replies 17 ·
Replies
17
Views
3K
  • · Replies 3 ·
Replies
3
Views
2K
  • · Replies 7 ·
Replies
7
Views
1K
  • · Replies 2 ·
Replies
2
Views
2K
  • · Replies 1 ·
Replies
1
Views
2K
  • · Replies 14 ·
Replies
14
Views
2K
  • · Replies 4 ·
Replies
4
Views
2K
Replies
22
Views
2K
  • · Replies 15 ·
Replies
15
Views
2K